Mobius strip computing unit normal field at 2 points

Click For Summary
The discussion focuses on computing the unit normal fields N1 and N2 for a Mobius band parametrization at specific points. The user is unsure how to start, particularly in substituting the given points into the parametrization F(s,t) to find the corresponding values of s and t. They have computed the partial derivatives dF/ds and dF/dt but are struggling with the cross product necessary for calculating the normal fields. The advice given emphasizes determining the appropriate parameters first to simplify the substitution process. Understanding the relationship between the parameters and the points is crucial for progressing with the calculations.
joeyjokester
Messages
3
Reaction score
0

Homework Statement


I am given this parametrization of the mobius band:
F(s,t) = (cos(t)+s*cos(t)*cos(t/2), sin(t)+s*sin(t)*cos(t/2), s*sin(t/2))
Let F1 be F restricted to (0,2*pi) X (-1,1).
Let F2 be F restricted to (-pi,pi) X (-1,1).
let N1 be the unit normal field determined by F1
let N2 be the unit normal field determined by F2

The question reads:
Compute the unit normal field for N1, N2 at the points (0,1,0) and (0,-1,0).

Homework Equations



N(t) = (dF/ds X dF/dt)/ ||dF/ds X dF/dt||


The Attempt at a Solution



I am unsure how to begin on this. I am looking for a starting block. Obviously I want to plug in the point before taking cross products? How do I do this when F(s,t) takes 2 parameters, but I am given a point of the form (x,y,z). This is troublesome. I have computed the partials dF/ds and dF/dt, but I don't know what to do with them.. I don't want to take this nasty cross product.

thanks
 
Physics news on Phys.org
take the derivatives to find the tangent vectors aligned with each parameter

then substitute in the given values & calculate the cross products, shoudln;t be too messy after substitution
 
That is exactly my question. How do I substitute in the given values?
 
you'll need to determine s & t for the given points first, to do that consider what would lead to the terms being zero
 
Question: A clock's minute hand has length 4 and its hour hand has length 3. What is the distance between the tips at the moment when it is increasing most rapidly?(Putnam Exam Question) Answer: Making assumption that both the hands moves at constant angular velocities, the answer is ## \sqrt{7} .## But don't you think this assumption is somewhat doubtful and wrong?

Similar threads

  • · Replies 3 ·
Replies
3
Views
1K
Replies
6
Views
2K
  • · Replies 8 ·
Replies
8
Views
2K
  • · Replies 31 ·
2
Replies
31
Views
4K
Replies
12
Views
2K
  • · Replies 18 ·
Replies
18
Views
2K
  • · Replies 8 ·
Replies
8
Views
2K
  • · Replies 105 ·
4
Replies
105
Views
6K
  • · Replies 3 ·
Replies
3
Views
2K
  • · Replies 10 ·
Replies
10
Views
4K